Gira con helicidad indefinida

Imagina que estamos estudiando la cuantización del espín a lo largo del mismo eje que el impulso. ¿Qué sucede si tengo un espinor de Dirac con un giro hacia arriba pero sin una helicidad definida ( ψ L , ψ R 0 ):

tu ( pag ) = ( pag · σ ξ pag · σ ¯ ξ ) = ( ψ L ψ R )

¿Cómo debo entender esto? Si el giro a lo largo de la pag -la dirección es positiva, ¿no debería la partícula tener una helicidad hacia la derecha definida y una componente hacia la izquierda de 0? En otras palabras, ¿no deberíamos tener tu ( pag ) ( 0 s o metro mi t h i norte gramo ) ?

Sí para una partícula sin masa. ¿Está haciendo una pregunta sobre la superposición parcial de la helicidad y la quiralidad en función de la masa?
De nada. Tampoco estoy preguntando sobre partículas sin masa en particular. Si el giro a lo largo de la pag -la dirección es positiva, ¿no debería la partícula tener una helicidad hacia la derecha definida y una componente hacia la izquierda de 0? es decir, no debería tu ( pag ) ( 0 s o metro mi t h i norte gramo ) ?
Sí, el giro alineado con el impulso denota helicidad positiva... No estoy seguro de por qué mencionas la lateralidad...
helicidad positiva ? ¿No querrás decir helicidad diestra ? También lamento no poder aclarar mi pregunta, pero no estoy seguro de lo que necesita que le dé más detalles.
Bien. Bueno, una partícula dextrógira (resp. zurda) significa que es un valor propio positivo (resp. negativo) del operador de helicidad.
Esto probablemente tiene que ver con el hecho de que la helicidad no es invariante de Lorentz; puede aumentar a un marco donde el impulso de la partícula apunta en la dirección opuesta. Sin embargo, la quiralidad es invariante de Lorentz.
Esto no tiene absolutamente nada que ver con mi pregunta.

Respuestas (3)

OK, sospecho que entiendo tu perplejidad... Simplemente estás engañado por el lenguaje descuidado e inapropiado "helicidad diestra". No existe tal cosa.

Para simplificar, tome el giro hacia arriba en la dirección z y el impulso en la dirección z , pag m = ( metro 2 + pag 2 , 0 , 0 , pag ) , e inspeccione su espinor en la base de Weyl (quiral), ψ L = 1 2 ( 1 γ 5 ) ψ = ( I 2 0 0 0 ) ψ , ψ R = 1 2 ( 1 + γ 5 ) ψ = ( 0 0 0 I 2 ) ψ ,

tu ( pag ) = ( pag · σ ξ pag · σ ¯ ξ ) = ( ψ L ψ R ) .

Ahora pag · σ ξ = mi pag   ξ , y pag · σ ¯ ξ = mi + pag   ξ , de modo que

tu ( pag ) = ( mi pag   ξ mi + pag   ξ ) ,
por supuesto tanto quiral izquierda como derecha. La componente superior no desaparece, en general, por helicidad positiva .

Inspeccione 3 límites. Para p = 0, E = m y

tu ( 0 ) = metro ( ξ ξ ) ,
completamente imparcial: la helicidad no está definida.

Para m = 0, los dos componentes superiores se proyectan y solo los dos inferiores (en realidad solo el tercero) sobreviven,

tu ( pag ) = 2 pag ( 0 ξ ) ,
tan verdaderamente diestro: el origen de referirse descuidadamente a la helicidad positiva como "R", ya que la helicidad y la quiralidad son idénticas.

Para pag metro ,

tu ( pag ) 2 pag ( metro / 2 pag   ξ ξ ) ,
así que también hay una pieza quiral izquierda, pero tremendamente subdominante a la pieza quiral derecha. Nuevamente, este es un espinor de helicidad positiva no divorciado de su pieza quiral izquierda.

La declaración del anverso se proporciona en Wikipedia : Para partículas masivas, distintos estados de quiralidad (p. ej., como ocurre en las cargas de interacción débiles) tienen componentes de helicidad tanto positivos como negativos, en proporciones proporcionales a la masa de la partícula .

El libro de texto de Itzykson & Zuber, 2-2-1 , proporciona una definición útil del operador de helicidad; en nuestro caso,

h ^ = 1 2 ( σ 3 0 0 σ 3 ) ,
de acuerdo con lo anterior: lee un valor propio de +1/2 tanto para el superior, ψ L , y el inferior, ψ R , componente, como debería.

A menos que uno esté seguro de evitar la trampa del lenguaje, o uno sea un fanático de las declaraciones paradójicas para mantener a la audiencia nerviosa, es una idea desafortunada estar lanzando términos de quiralidad para cuantificar la helicidad...

  • PD. ¡Un título más apropiado habría sido "Helicity con quiralidad indefinida"!
Gracias por tus esfuerzos. Puede ser por mi falta de inteligencia, pero no siento que esto responda mi pregunta en absoluto. Simplemente, si el espín está alineado a lo largo pag ¿No debería el espinor superior en tu ( pag ) ser 0 ? Por cierto, creo que podría consultar esta otra pregunta que publiqué también en physics.stackexchange.com/questions/371978/… , está algo relacionado.
No. No. No. Te muestro por qué y cómo. Por favor revise su quiralidad y no la confunda con helictidad.
De nuevo con quiralidad. Te agradezco mucho que intentes ayudarme y no pretendo sonar grosero en absoluto pero sigues insistiendo en 1 cosa y te niegas a entender lo que digo. Es como si me estuvieras diciendo ''no no, esta no es tu pregunta, tu pregunta es esta ''. No, sé lo que quiero preguntar.

Hay dos cosas que creo que podrías estar malinterpretando. Una es que el espinor no solo contiene información sobre el giro, también contiene información sobre la energía y el momento.

tu ¯ γ m tu = 2 ( mi , pag )
Necesitas usar los 4 componentes del espinor para describir esto.

La otra cosa es llamar a los componentes de un espinor ψ L y ψ R solo tiene sentido para partículas sin masa (o partículas que se mueven muy rápido).

Si estás en reposo, pag = ( metro , 0 , 0 , 0 ) y el componente 0 de σ es la matriz identidad, por lo que su espinor es simplemente

tu ( pag r mi s t F r a metro mi ) = ( metro ξ metro ξ )
Y ambos ψ L = ψ R no importa si está apuntando hacia arriba o hacia abajo (o cualquier punto intermedio). A medida que impulsa los componentes en la dirección del giro, uno de ψ L o ψ R se hará más pequeño y el otro más grande. En el límite de momento muy grande pero finito, una de estas dos componentes será muy pequeña dependiendo de la helicidad. Este es el límite en el que los nombres ψ R y ψ L tener sentido. Pero el componente 'incorrecto' aún debe ser distinto de cero para brindarle información sobre el impulso.

Entonces el problema es que no entendí eso ψ L y ψ R son estados propios de quiralidad , no estados propios de helicidad .

En el límite de alta energía donde la helicidad y la quiralidad son básicamente lo mismo, el estado de hecho se convierte en un estado propio (hice los cálculos, pero es demasiado largo para publicarlo aquí a menos que alguien lo solicite).